Functions without Laplace transform?












1












$begingroup$


We have just started working with Laplace transformations at our university course. One of the I came across as following:



Provide three examples of functions for which the Laplace transform does not exist.



We use the book written by Kreyszig "Advanced Engineering Mathematics".



I would have answered where $S > 0$ and $S > K$, I am not sure what the third kind of function would be. In addition I am unsure if this is even the answer they are looking for, I am hoping someone more experienced could provide me with some suggestions. Worst case scenario I get my answer in two weeks time when I hand in the assignment :)










share|cite|improve this question











$endgroup$








  • 3




    $begingroup$
    Laplace transform for $f(t)$ works if $int_0^infty e^{-st} f(t) dt$ converges. So if your function grows so fast that no decaying exponential can stop it then the integral diverges. Think about fast growing functions, even faster than any $e^{at}$ to find an example.
    $endgroup$
    – Maesumi
    Feb 24 '13 at 17:07
















1












$begingroup$


We have just started working with Laplace transformations at our university course. One of the I came across as following:



Provide three examples of functions for which the Laplace transform does not exist.



We use the book written by Kreyszig "Advanced Engineering Mathematics".



I would have answered where $S > 0$ and $S > K$, I am not sure what the third kind of function would be. In addition I am unsure if this is even the answer they are looking for, I am hoping someone more experienced could provide me with some suggestions. Worst case scenario I get my answer in two weeks time when I hand in the assignment :)










share|cite|improve this question











$endgroup$








  • 3




    $begingroup$
    Laplace transform for $f(t)$ works if $int_0^infty e^{-st} f(t) dt$ converges. So if your function grows so fast that no decaying exponential can stop it then the integral diverges. Think about fast growing functions, even faster than any $e^{at}$ to find an example.
    $endgroup$
    – Maesumi
    Feb 24 '13 at 17:07














1












1








1





$begingroup$


We have just started working with Laplace transformations at our university course. One of the I came across as following:



Provide three examples of functions for which the Laplace transform does not exist.



We use the book written by Kreyszig "Advanced Engineering Mathematics".



I would have answered where $S > 0$ and $S > K$, I am not sure what the third kind of function would be. In addition I am unsure if this is even the answer they are looking for, I am hoping someone more experienced could provide me with some suggestions. Worst case scenario I get my answer in two weeks time when I hand in the assignment :)










share|cite|improve this question











$endgroup$




We have just started working with Laplace transformations at our university course. One of the I came across as following:



Provide three examples of functions for which the Laplace transform does not exist.



We use the book written by Kreyszig "Advanced Engineering Mathematics".



I would have answered where $S > 0$ and $S > K$, I am not sure what the third kind of function would be. In addition I am unsure if this is even the answer they are looking for, I am hoping someone more experienced could provide me with some suggestions. Worst case scenario I get my answer in two weeks time when I hand in the assignment :)







examples-counterexamples laplace-transform






share|cite|improve this question















share|cite|improve this question













share|cite|improve this question




share|cite|improve this question








edited Apr 30 '18 at 10:30









GNUSupporter 8964民主女神 地下教會

13.3k72549




13.3k72549










asked Feb 24 '13 at 17:00









WobbleyWobbley

12314




12314








  • 3




    $begingroup$
    Laplace transform for $f(t)$ works if $int_0^infty e^{-st} f(t) dt$ converges. So if your function grows so fast that no decaying exponential can stop it then the integral diverges. Think about fast growing functions, even faster than any $e^{at}$ to find an example.
    $endgroup$
    – Maesumi
    Feb 24 '13 at 17:07














  • 3




    $begingroup$
    Laplace transform for $f(t)$ works if $int_0^infty e^{-st} f(t) dt$ converges. So if your function grows so fast that no decaying exponential can stop it then the integral diverges. Think about fast growing functions, even faster than any $e^{at}$ to find an example.
    $endgroup$
    – Maesumi
    Feb 24 '13 at 17:07








3




3




$begingroup$
Laplace transform for $f(t)$ works if $int_0^infty e^{-st} f(t) dt$ converges. So if your function grows so fast that no decaying exponential can stop it then the integral diverges. Think about fast growing functions, even faster than any $e^{at}$ to find an example.
$endgroup$
– Maesumi
Feb 24 '13 at 17:07




$begingroup$
Laplace transform for $f(t)$ works if $int_0^infty e^{-st} f(t) dt$ converges. So if your function grows so fast that no decaying exponential can stop it then the integral diverges. Think about fast growing functions, even faster than any $e^{at}$ to find an example.
$endgroup$
– Maesumi
Feb 24 '13 at 17:07










4 Answers
4






active

oldest

votes


















1












$begingroup$

Hint: The function $e^{e^t}$ could be an answer?
Aside: Bilateral Laplace transform of $e^{-e^{-t}}$ is Gamma function?






share|cite|improve this answer











$endgroup$





















    1












    $begingroup$

    Find something that grows too fast for the exponentially-decaying damping factor $e^{-st}$ (where $t$ is the variable of integration) in the defining integral of the transform to cancel. What do you know of that grows more quickly than an exponential function?






    share|cite|improve this answer









    $endgroup$





















      1












      $begingroup$

      Find some functions that exist $limlimits_{tto a}f(t)$ diverges for some positive real numbers $a$ , e.g. $tan t$ .






      share|cite|improve this answer









      $endgroup$





















        0












        $begingroup$

        $$F(t)=frac{1}{t}$$
        Laplace transformation of this function does not exists






        share|cite|improve this answer











        $endgroup$













          Your Answer





          StackExchange.ifUsing("editor", function () {
          return StackExchange.using("mathjaxEditing", function () {
          StackExchange.MarkdownEditor.creationCallbacks.add(function (editor, postfix) {
          StackExchange.mathjaxEditing.prepareWmdForMathJax(editor, postfix, [["$", "$"], ["\\(","\\)"]]);
          });
          });
          }, "mathjax-editing");

          StackExchange.ready(function() {
          var channelOptions = {
          tags: "".split(" "),
          id: "69"
          };
          initTagRenderer("".split(" "), "".split(" "), channelOptions);

          StackExchange.using("externalEditor", function() {
          // Have to fire editor after snippets, if snippets enabled
          if (StackExchange.settings.snippets.snippetsEnabled) {
          StackExchange.using("snippets", function() {
          createEditor();
          });
          }
          else {
          createEditor();
          }
          });

          function createEditor() {
          StackExchange.prepareEditor({
          heartbeatType: 'answer',
          autoActivateHeartbeat: false,
          convertImagesToLinks: true,
          noModals: true,
          showLowRepImageUploadWarning: true,
          reputationToPostImages: 10,
          bindNavPrevention: true,
          postfix: "",
          imageUploader: {
          brandingHtml: "Powered by u003ca class="icon-imgur-white" href="https://imgur.com/"u003eu003c/au003e",
          contentPolicyHtml: "User contributions licensed under u003ca href="https://creativecommons.org/licenses/by-sa/3.0/"u003ecc by-sa 3.0 with attribution requiredu003c/au003e u003ca href="https://stackoverflow.com/legal/content-policy"u003e(content policy)u003c/au003e",
          allowUrls: true
          },
          noCode: true, onDemand: true,
          discardSelector: ".discard-answer"
          ,immediatelyShowMarkdownHelp:true
          });


          }
          });














          draft saved

          draft discarded


















          StackExchange.ready(
          function () {
          StackExchange.openid.initPostLogin('.new-post-login', 'https%3a%2f%2fmath.stackexchange.com%2fquestions%2f313056%2ffunctions-without-laplace-transform%23new-answer', 'question_page');
          }
          );

          Post as a guest















          Required, but never shown

























          4 Answers
          4






          active

          oldest

          votes








          4 Answers
          4






          active

          oldest

          votes









          active

          oldest

          votes






          active

          oldest

          votes









          1












          $begingroup$

          Hint: The function $e^{e^t}$ could be an answer?
          Aside: Bilateral Laplace transform of $e^{-e^{-t}}$ is Gamma function?






          share|cite|improve this answer











          $endgroup$


















            1












            $begingroup$

            Hint: The function $e^{e^t}$ could be an answer?
            Aside: Bilateral Laplace transform of $e^{-e^{-t}}$ is Gamma function?






            share|cite|improve this answer











            $endgroup$
















              1












              1








              1





              $begingroup$

              Hint: The function $e^{e^t}$ could be an answer?
              Aside: Bilateral Laplace transform of $e^{-e^{-t}}$ is Gamma function?






              share|cite|improve this answer











              $endgroup$



              Hint: The function $e^{e^t}$ could be an answer?
              Aside: Bilateral Laplace transform of $e^{-e^{-t}}$ is Gamma function?







              share|cite|improve this answer














              share|cite|improve this answer



              share|cite|improve this answer








              edited Apr 8 '13 at 7:36









              robjohn

              268k27308633




              268k27308633










              answered Apr 8 '13 at 6:17









              Kannan NambiarKannan Nambiar

              111




              111























                  1












                  $begingroup$

                  Find something that grows too fast for the exponentially-decaying damping factor $e^{-st}$ (where $t$ is the variable of integration) in the defining integral of the transform to cancel. What do you know of that grows more quickly than an exponential function?






                  share|cite|improve this answer









                  $endgroup$


















                    1












                    $begingroup$

                    Find something that grows too fast for the exponentially-decaying damping factor $e^{-st}$ (where $t$ is the variable of integration) in the defining integral of the transform to cancel. What do you know of that grows more quickly than an exponential function?






                    share|cite|improve this answer









                    $endgroup$
















                      1












                      1








                      1





                      $begingroup$

                      Find something that grows too fast for the exponentially-decaying damping factor $e^{-st}$ (where $t$ is the variable of integration) in the defining integral of the transform to cancel. What do you know of that grows more quickly than an exponential function?






                      share|cite|improve this answer









                      $endgroup$



                      Find something that grows too fast for the exponentially-decaying damping factor $e^{-st}$ (where $t$ is the variable of integration) in the defining integral of the transform to cancel. What do you know of that grows more quickly than an exponential function?







                      share|cite|improve this answer












                      share|cite|improve this answer



                      share|cite|improve this answer










                      answered Apr 8 '13 at 7:39









                      The_SympathizerThe_Sympathizer

                      7,7102245




                      7,7102245























                          1












                          $begingroup$

                          Find some functions that exist $limlimits_{tto a}f(t)$ diverges for some positive real numbers $a$ , e.g. $tan t$ .






                          share|cite|improve this answer









                          $endgroup$


















                            1












                            $begingroup$

                            Find some functions that exist $limlimits_{tto a}f(t)$ diverges for some positive real numbers $a$ , e.g. $tan t$ .






                            share|cite|improve this answer









                            $endgroup$
















                              1












                              1








                              1





                              $begingroup$

                              Find some functions that exist $limlimits_{tto a}f(t)$ diverges for some positive real numbers $a$ , e.g. $tan t$ .






                              share|cite|improve this answer









                              $endgroup$



                              Find some functions that exist $limlimits_{tto a}f(t)$ diverges for some positive real numbers $a$ , e.g. $tan t$ .







                              share|cite|improve this answer












                              share|cite|improve this answer



                              share|cite|improve this answer










                              answered Apr 8 '13 at 8:33









                              doraemonpauldoraemonpaul

                              12.6k31660




                              12.6k31660























                                  0












                                  $begingroup$

                                  $$F(t)=frac{1}{t}$$
                                  Laplace transformation of this function does not exists






                                  share|cite|improve this answer











                                  $endgroup$


















                                    0












                                    $begingroup$

                                    $$F(t)=frac{1}{t}$$
                                    Laplace transformation of this function does not exists






                                    share|cite|improve this answer











                                    $endgroup$
















                                      0












                                      0








                                      0





                                      $begingroup$

                                      $$F(t)=frac{1}{t}$$
                                      Laplace transformation of this function does not exists






                                      share|cite|improve this answer











                                      $endgroup$



                                      $$F(t)=frac{1}{t}$$
                                      Laplace transformation of this function does not exists







                                      share|cite|improve this answer














                                      share|cite|improve this answer



                                      share|cite|improve this answer








                                      edited Apr 30 '18 at 9:53









                                      Alex Vong

                                      1,309819




                                      1,309819










                                      answered Apr 30 '18 at 9:25









                                      RKMVRKMV

                                      1




                                      1






























                                          draft saved

                                          draft discarded




















































                                          Thanks for contributing an answer to Mathematics Stack Exchange!


                                          • Please be sure to answer the question. Provide details and share your research!

                                          But avoid



                                          • Asking for help, clarification, or responding to other answers.

                                          • Making statements based on opinion; back them up with references or personal experience.


                                          Use MathJax to format equations. MathJax reference.


                                          To learn more, see our tips on writing great answers.




                                          draft saved


                                          draft discarded














                                          StackExchange.ready(
                                          function () {
                                          StackExchange.openid.initPostLogin('.new-post-login', 'https%3a%2f%2fmath.stackexchange.com%2fquestions%2f313056%2ffunctions-without-laplace-transform%23new-answer', 'question_page');
                                          }
                                          );

                                          Post as a guest















                                          Required, but never shown





















































                                          Required, but never shown














                                          Required, but never shown












                                          Required, but never shown







                                          Required, but never shown

































                                          Required, but never shown














                                          Required, but never shown












                                          Required, but never shown







                                          Required, but never shown







                                          Popular posts from this blog

                                          How do I know what Microsoft account the skydrive app is syncing to?

                                          When does type information flow backwards in C++?

                                          Grease: Live!